4
$\begingroup$

Let $L \in R$ be an infinite recursive language. Is it always possible to find a subset $L' \subseteq L$ such that $L' \in \mathbb{RE} \setminus R$?
In other words, can every infinite decidable language contain a subset that is recursively enumerable but undecidable?

I'm interested in both a general proof and any construction ideas or examples of such a subset $L'$.

asked Jun 3 at 10:53
$\endgroup$

1 Answer 1

3
$\begingroup$

As $L$ is infinite and is a subset of words, namely $L\subseteq \Sigma_L^*$ for some finite alphabet $\Sigma_L$, then $|L| = \aleph_0$. Therefore, you can encode the Halting-problem using words in $L$. More formally, let $A$ be an alphabet encoding the Halting-problem. Thus, the Halting-problem $HP_A$ is a subset of $A^*$. Next, as $L$ is decidable, then there is a Turing machine $M_{A^* \to L}$ that computes a bijection $\eta: A^* \to L$. Indeed, given input $x$ for $M_{A^* \to L}$, the machine $M_{A^* \to L}$ can go over all the words in $\Sigma^*_L$ in min-lex order (we order first according to length, and then lexicographically), and compute the word in $L$ corresponding to $x$ according to a min-lex order on $A^*$. Note that the latter can be done as $L$ is decidable, so eventually $M_{A^* \to L}$ finds the word in $L$ corresponding to $x$. Specifically, if $x$ is the $i$’th word in $A^*$ according to the min-lex order on $A^*$, then $\eta(x)$ is the $i$’th word in $L$ according to the min-lex order on $\Sigma^*_L$.

Now consider the language $HP_L = \{ y: \eta^{-1}(y) \in HP_A \} \subseteq L \subseteq \Sigma_L^*$. It is not hard to see that $M_{A^* \to L} $ computes a mapping reduction from $HP_A$ to $HP_L$, and thus $HP_L$ is not decidable. On the other hand, $HP_L$ is in $\text{RE}$ because given any word $y$ over $\Sigma_L$, we can first check if $y \in L$ (since $L$ is decidable), and then check whether $\eta^{-1}(y)\in HP_A$ by first computing $\eta^{-1}(y)$ (similarly to how $M_{A^* \to L}$ works) and then simulate a machine that recognizes $HP_A$ on $\eta^{-1}(y)$.

answered Jun 3 at 13:50
$\endgroup$
4
  • $\begingroup$ Thanks for the answer, unfortunately I haven't learnt some of the symbols you used here, suppose I'm attending the first computability course, is there a simpler proof ? $\endgroup$ Commented Jun 3 at 19:46
  • 3
    $\begingroup$ @csmathstudent8 Intuitively, the point is that as long as you have an infinite list of numbers, you can "pretend" that these numbers are $\mathbb N = \{1, 2, 3, ...\},ドル because there must be a bijective function $\eta$ (eta) from $L$ to $\mathbb N$. But then you can take your favorite undecidable problem, e.g. HALT, and "remap" the values using $\eta$. $\endgroup$ Commented Jun 4 at 9:19
  • $\begingroup$ @csmathstudent8 The underlying idea is shown in the comment above. If there are questions about some notations, you are welcome to ask. I am not sure if there is a much simpler proof in terms of used symbols. $\endgroup$ Commented Jun 4 at 15:40
  • $\begingroup$ I understood now the recap you wrote above, but yes I guess it's currently above my course requirements. $\endgroup$ Commented Jun 5 at 9:31

Your Answer

Draft saved
Draft discarded

Sign up or log in

Sign up using Google
Sign up using Email and Password

Post as a guest

Required, but never shown

Post as a guest

Required, but never shown

By clicking "Post Your Answer", you agree to our terms of service and acknowledge you have read our privacy policy.

Start asking to get answers

Find the answer to your question by asking.

Ask question

Explore related questions

See similar questions with these tags.